Please confirm topic selection

Are you sure you want to trigger topic in your Anconeus AI algorithm?

Please confirm action

You are done for today with this topic.

Would you like to start learning session with this topic items scheduled for future?

Review Question - QID 210561

In scope icon N/A C
QID 210561 (Type "210561" in App Search)
A 67-year-old man presents to his primary care physician for a wellness checkup. The patient has a past medical history of hypertension and diabetes and is currently taking metformin, insulin, and lisinopril. He lives alone, drinks 7 to 10 alcoholic beverages per day, and smokes 3 packs of cigarettes per day. His temperature is 98.6°F (37.0°C), blood pressure is 177/108 mmHg, pulse is 90/min, respirations are 16/min, and oxygen saturation is 99% on room air. Physical exam demonstrates new findings as compared to his appointment 1 month ago, which include a systolic murmur best heard in the right upper sternal area, the findings in Figure A, and an enlarged, soft, and symmetric prostate. Which of the following is associated with the most likely diagnosis?
  • A
  • A
Add Colleague
Lab Values
Calculator